Last visit was: 23 Apr 2024, 22:40 It is currently 23 Apr 2024, 22:40

Close
GMAT Club Daily Prep
Thank you for using the timer - this advanced tool can estimate your performance and suggest more practice questions. We have subscribed you to Daily Prep Questions via email.

Customized
for You

we will pick new questions that match your level based on your Timer History

Track
Your Progress

every week, we’ll send you an estimated GMAT score based on your performance

Practice
Pays

we will pick new questions that match your level based on your Timer History
Not interested in getting valuable practice questions and articles delivered to your email? No problem, unsubscribe here.
Close
Request Expert Reply
Confirm Cancel
SORT BY:
Date
Tags:
Show Tags
Hide Tags
User avatar
Intern
Intern
Joined: 16 Oct 2010
Posts: 4
Own Kudos [?]: 12 [1]
Given Kudos: 2
Send PM
avatar
Intern
Intern
Joined: 08 Oct 2010
Posts: 3
Own Kudos [?]: [0]
Given Kudos: 1
Send PM
User avatar
Retired Moderator
Joined: 02 Sep 2010
Posts: 615
Own Kudos [?]: 2929 [0]
Given Kudos: 25
Location: London
 Q51  V41
Send PM
User avatar
Manager
Manager
Joined: 16 Oct 2010
Posts: 65
Own Kudos [?]: 177 [1]
Given Kudos: 3
Location: United States
Concentration: Finance, Entrepreneurship
GMAT 1: 700 Q49 V35
WE:Information Technology (Investment Banking)
Send PM
Re: Geomtery, circles [#permalink]
1
Kudos
honeyhani wrote:
In the figure below, AB is the chord of a circle with center O. AB is extended to C such that BC = OB. The straight line CO is produced to meet the circle at D. If ACD= y degrees and AOD = x degrees such that x = ky, then the value of k is

A. 3
B. 2
C. 1
D. None of the above

please help me with answers in details.


Sol:
OB = BC (given) => Angle BOC = Angle BCO = y => Angle OBC = 180-2y (in triangle OBC)
Hence Angle OBA = 2y
Since AO = OB (Both radius of the circle)
so Angle OBA = Angle BAO = 2y => Angle AOB = 180 -4y
Now we know all the three angles at point O form by a straight line DC
Hence Angle AOD + Angle AOB + Angle BOC = 180
x + 180 - 4y + y = 180
x - 3y = 0
x = 3y
Answer (A)
Manager
Manager
Joined: 18 Jun 2018
Posts: 222
Own Kudos [?]: 415 [0]
Given Kudos: 35
Send PM
Re: In the figure below, AB is the chord of a circle with center [#permalink]
honeyhani wrote:
In the figure below, AB is the chord of a circle with center O. AB is extended to C such that BC = OB. The straight line CO is produced to meet the circle at D. If ACD= y degrees and AOD = x degrees such that x = ky, then the value of k is

A. 3
B. 2
C. 1
D. None of the above

please help me with answers in details.


OA:A
Attachment:
xy.PNG
xy.PNG [ 54.81 KiB | Viewed 8554 times ]


Joining \(OB\),\(\angle BOC =\angle BCO =y\) (As \(OB=BC\))
\(\angle OBA = \angle BOC + \angle BCO\) (An exterior angle of a triangle is equal to the sum of the opposite interior angles.)
\(\angle OBA = 2y\)
\(\angle OAB = \angle OBA =2y\) ( As \(OA=OB=r\))
Considering \(\triangle OAC\),
\(\angle DOA =\angle OAC + \angle OCA\) (An exterior angle of a triangle is equal to the sum of the opposite interior angles.)
\(x =y +2y\)
\(x=3y\) so \(k=3\)
Tutor
Joined: 04 Aug 2010
Posts: 1315
Own Kudos [?]: 3134 [0]
Given Kudos: 9
Schools:Dartmouth College
Send PM
In the figure below, AB is the chord of a circle with center [#permalink]
Expert Reply
Quote:
In the figure below, AB is the chord of a circle with center O. AB is extended to C such that BC = OB. The straight line CO is produced to meet the circle at D. If ACD= y degrees and AOD = x degrees such that x = ky, then the value of k is

A. 3
B. 2
C. 1
D. None of the above


OA and OB are radii, so OA=OB.
Since the prompt indicates that OB=BC, we get:
OA=OB=BC.
The following figure is yielded:


Let y=10.
Since the angles inside triangle BCO must sum to 180, and the angles opposite sides OB and BC must be equal, the following figure is yielded:


Since angles ABO and OBC must sum to 180, angle ABO=20.
Since the angles inside triangle ABO must sum to 180, and the angles opposite sides OA and OB must be equal, the following figure is yielded:


Since angles AOD, AOB and BOC must sum to 180, we get:


Since x=30, y=10, and \(k = \frac{x}{y}\), we get:
\(k = \frac{30}{10} = 3\)

GMAT Club Bot
In the figure below, AB is the chord of a circle with center [#permalink]
Moderators:
Math Expert
92883 posts
Senior Moderator - Masters Forum
3137 posts

Powered by phpBB © phpBB Group | Emoji artwork provided by EmojiOne